Difference between revisions of "2016 UNCO Math Contest II Problems/Problem 7"

(Problem)
(Solution 2)
(3 intermediate revisions by 2 users not shown)
Line 4: Line 4:
  
 
== Solution ==
 
== Solution ==
 +
<math>\frac{5}{4}</math>
 +
 +
==Solution 2==
 +
 +
 +
This is a telescoping series:
 +
 +
(1−1/9)+(1/4−1/16)+(1/9−1/25)+(1/16−1/36)+(1/25−1/49)+...=5/4
  
 
== See also ==
 
== See also ==
 
{{UNCO Math Contest box|year=2016|n=II|num-b=6|num-a=8}}
 
{{UNCO Math Contest box|year=2016|n=II|num-b=6|num-a=8}}
  
[[Category:]]
+
[[Category: Intermediate Algebra Problems]]

Revision as of 20:18, 4 March 2021

Problem

Evaluate \[S =\sum_{n=2}^{\infty} \frac{4n}{(n^2-1)^2}\]

Solution

$\frac{5}{4}$

Solution 2

This is a telescoping series:

(1−1/9)+(1/4−1/16)+(1/9−1/25)+(1/16−1/36)+(1/25−1/49)+...=5/4

See also

2016 UNCO Math Contest II (ProblemsAnswer KeyResources)
Preceded by
Problem 6
Followed by
Problem 8
1 2 3 4 5 6 7 8 9 10
All UNCO Math Contest Problems and Solutions